Proving Closedness of a Set in a Metric Space

In summary: Well you got to first decide if you want to continue trying to prove that the compliment... or just accept that the set is closed.
  • #1
Lee33
160
0

Homework Statement



Prove that if lim n→∞ (p_n ) = p in a metric space then the set of points {p,p_1,p_2, ...,} are closed.

2. Relevant information

The definition of close in my book is "a set is closed if and only if its complementary is open." So I want to prove this by contradiction. I can't prove it by using accumulation points or compactness.

A theorem in my book states that a set S in a metric space is closed if and only if whenever {q_1,q_2 ,... } is a sequence of points in S that is convergent then the lim n→∞ q_n ∈S .

The Attempt at a Solution



Suppose the set S is not closed which will imply that S^c is not open then there exists a point in S^c; let's denote this point p ∈ S^c such that there is an open ball B(p,r)∈ S^c ...

But I can't continue any further because I don't know what to do next.
 
Last edited:
Physics news on Phys.org
  • #2
I have a feeling this problem is not stated quite right. They haven't given you any way to determine whether that limit is in the space S or not.

Here's an example where it doesn't work: Let Q be the metric space of rational numbers where the metric is usual distance |x - y|. Let ##p_n## be a sequence of rational numbers whose limit is ## \sqrt 2##. Clearly the limit is not in Q and the sequence is not a closed set.

If you were told the metric space is complete, you would have a chance.
 
  • #3
If that is the case then I am sure that the metric space must be complete then.
 
  • #4
brmath said:
I have a feeling this problem is not stated quite right. They haven't given you any way to determine whether that limit is in the space S or not.

Here's an example where it doesn't work: Let Q be the metric space of rational numbers where the metric is usual distance |x - y|. Let ##p_n## be a sequence of rational numbers whose limit is ## \sqrt 2##. Clearly the limit is not in Q and the sequence is not a closed set.

If you were told the metric space is complete, you would have a chance.

In the metric space ##\mathbb{Q}##, a sequence ##p_n## of positive elements such that ##p_n^2\rightarrow 2## does not have a limit. It's not that there is a limit, just not in ##\mathbb{Q}##; there isn't a limit. Such a sequence is closed in ##\mathbb{Q}##; every element of ##\mathbb{Q}## not in the sequence has an open neighborhood which does not contain any elements of the sequence.
 
  • #5
Lee33 said:

Homework Statement



Prove that if lim n→∞ (p_n ) = p in a metric space then the set of points {p,p_1,p_2, ...,} are closed.

2. Relevant information

The definition of close in my book is "a set is closed if and only if its complementary is open." So I want to prove this by contradiction. I can't prove it by using accumulation points or compactness.

A theorem in my book states that a set S in a metric space is closed if and only if whenever {q_1,q_2 ,... } is a sequence of points in S that is convergent then the lim n→∞ q_n ∈S .

The Attempt at a Solution



Suppose the set S is not closed which will imply that S^c is not open then there exists a point in S^c; let's denote this point p ∈ S^c such that there is an open ball B(p,r)∈ S^c ...

But I can't continue any further because I don't know what to do next.

You want to use that theorem in your book. What kinds of sequences in your set can converge? What are the limit points of your set?
 
  • #6
gopher_p said:
You want to use that theorem in your book. What kinds of sequences in your set can converge? What are the limit points of your set?

Sequence that can converge are bounded. The limit point will just be the point p because limit points are unique.
 
  • #7
Lee33 said:
Sequence that can converge are bounded. The limit point will just be the point p because limit points are unique.

What about the sequence ##p_1,p_1,p_1,...##?
 
  • #8
They are in our set S.
 
  • #9
Does the sequence ##p_1,p_1,p_1,...## converge? To what?
 
  • #10
gopher_p said:
Does the sequence ##p_1,p_1,p_1,...## converge? To what?

Yes the sequence ##p_1,p_1,p_1,...## does converge to the point p.
 
  • #11
Lee33 said:
Yes the sequence ##p_1,p_1,p_1,...## does converge to the point p.


##p_1,p_1,p_1,...## is a constant sequence. It converges to ##p_1##.
 
  • #12
I didn't carefully look at your sequence. I thought it was p1, p2, ... pn. But yes it converges to p1, but then how can I finish my proof?
 
  • #13
Lee33 said:
I didn't carefully look at your sequence. I thought it was p1, p2, ... pn. But yes it converges to p1, but then how can I finish my proof?

Well you got to first decide if you want to continue trying to prove that the compliment of your set is open or if you want to take my advice an use the theorem from your book. If you want to use the theorem from your book, then you need to think about the types of sequences that you can get from the set ##\{ p,p_1,p_2,...\}## and the potential limits of those sequences. One further hint that I'll give is to consider convergent sequences which contain only finitely many distinct elements from your set and then those that contain infinitely many distinct elements.

Also, and I can only speculate, but you might find some inspiration in the proof of the theorem from your book. You might be able to borrow some ideas from that proof and use them in yours.
 
  • #14
If you want to use the complement of a closed set is open definition, then I would do proof by contradiction. Suppose the complement is not open, and you have a point q which is not p or p1, p2,..., and for every [itex] \epsilon[/itex], there exists some n such that [itex] d(q,p_n) \leq \epsilon [/itex].

Can you derive a contradiction from this?
 
  • #15
gopher_p said:
In the metric space ##\mathbb{Q}##, a sequence ##p_n## of positive elements such that ##p_n^2\rightarrow 2## does not have a limit. It's not that there is a limit, just not in ##\mathbb{Q}##; there isn't a limit. Such a sequence is closed in ##\mathbb{Q}##; every element of ##\mathbb{Q}## not in the sequence has an open neighborhood which does not contain any elements of the sequence.

You certainly are right -- thank you for pointing it out to me.
 
1.

What is a closed set in a metric space?

A closed set in a metric space is a set that contains all its limit points. In other words, if a sequence of points in the set converges to a point, that point must also be in the set. This concept is important in topology and analysis, as closed sets have many useful properties.

2.

How do you prove that a set is closed in a metric space?

To prove that a set is closed in a metric space, you can use the definition of a closed set and show that it contains all its limit points. This can be done by assuming a point is a limit point of the set and showing that it is also in the set. Another method is to show that the complement of the set (the set of points not in the set) is open.

3.

What is the importance of proving closedness in a metric space?

Proving closedness in a metric space is important because it allows us to determine whether a set has certain properties, such as compactness or connectedness. It also helps us in proving the continuity of functions and in understanding the behavior of sequences and convergence.

4.

Are there any common techniques for proving closedness in a metric space?

Yes, there are several common techniques for proving closedness in a metric space. One technique is to use the definition of a closed set and show that it contains all its limit points. Another technique is to use the fact that the complement of a closed set is open. Additionally, you can use the properties of open sets, such as the union and intersection of open sets being open.

5.

Can a set be both open and closed in a metric space?

Yes, a set can be both open and closed in a metric space. This is known as a clopen set, and it is only possible in certain spaces, such as discrete metric spaces. In most metric spaces, a set cannot be both open and closed, except for the empty set and the entire space.

Similar threads

  • Calculus and Beyond Homework Help
Replies
3
Views
503
  • Calculus and Beyond Homework Help
Replies
1
Views
1K
Replies
1
Views
1K
  • Calculus and Beyond Homework Help
Replies
8
Views
2K
  • Calculus and Beyond Homework Help
Replies
1
Views
5K
  • Calculus and Beyond Homework Help
Replies
5
Views
1K
  • Calculus and Beyond Homework Help
Replies
1
Views
1K
  • Calculus and Beyond Homework Help
Replies
7
Views
1K
  • Calculus and Beyond Homework Help
Replies
15
Views
1K
Replies
5
Views
1K
Back
Top